Evaluating Triple Integrals on a Sphere in the First Octant

Click For Summary
The discussion focuses on evaluating triple integrals of the function z^2 over a sphere defined by x^2 + y^2 + z^2 = a^2 in the first octant. For part ii), the integration limits are established as 0 ≤ r < a, 0 ≤ φ < π/2, and 0 ≤ θ < π/2, leading to a calculated integral result of r^5/30 * π. However, part i) raises confusion regarding the interpretation of the integral's limits, as it seems to imply evaluating only on the surface of the sphere, which contradicts the nature of a triple integral. The consensus is that part i) likely refers to the interior of the sphere in the first octant, clarifying the initial misunderstanding. This highlights the importance of correctly interpreting the problem statement for accurate evaluation.
amiras
Messages
64
Reaction score
0

Homework Statement



Evaluate triple integral

z^2 dxdydz

throughout
i) the part of the sphere x^2 + y^2 + z^2 = a^2 (first octant)
ii)the complete interior of the sphere x^2 + y^2 + z^2 = a^2 (first octant)


Homework Equations



It is probably good idea to work in spherical coords.

z = r*cosφ
x = r*sinφ cosθ
y = r*sinφ sinθ

dxdydz = r^2 sinφ drdφdθ

The Attempt at a Solution



I'l start at part ii) because its the part I can do.
Here the boundaries are:
0 =< r < a
0 =< φ < pi/2
0 =< θ < pi/2

the integration now becomes:

(Int[r=0, a] r^4 dr )( Int[φ=0, pi/2] sinφcos^2 φ)( Int [θ=0, pi/2]) = r^5/30 * pi

i) But for part i), I am confused. The integral should be evaluated only on the surface of the sphere. The radius a is constant in length, so how should r be defined?
a < r < a, makes no sense.

Need advice.
 
Last edited:
Physics news on Phys.org
Are you sure the problem is quoted correctly?
 
Part i obviously means the interior of the sphere in the first octant. Otherwise it wouldn't be a triple integral.
 
You are right, I understand what nonsense I was thinking about. The part i) is not quoted correctly. Thanks!
 
Question: A clock's minute hand has length 4 and its hour hand has length 3. What is the distance between the tips at the moment when it is increasing most rapidly?(Putnam Exam Question) Answer: Making assumption that both the hands moves at constant angular velocities, the answer is ## \sqrt{7} .## But don't you think this assumption is somewhat doubtful and wrong?

Similar threads

  • · Replies 9 ·
Replies
9
Views
2K
  • · Replies 2 ·
Replies
2
Views
2K
  • · Replies 19 ·
Replies
19
Views
2K
  • · Replies 3 ·
Replies
3
Views
2K
  • · Replies 4 ·
Replies
4
Views
2K
  • · Replies 9 ·
Replies
9
Views
1K
  • · Replies 21 ·
Replies
21
Views
3K
Replies
3
Views
2K
  • · Replies 2 ·
Replies
2
Views
2K
Replies
4
Views
2K